Sr Examen

Otras calculadoras

  • ¿Cómo usar?

  • Suma de la serie:
  • (4/7)^n (4/7)^n
  • (4^n-3^n)/12^n (4^n-3^n)/12^n
  • 1/n^5 1/n^5
  • ((((3.7)^n)(n^2))/((2.1)^n))((x-3)^n)
  • Expresiones idénticas

  • ((((tres . siete)^n)(n^ dos))/((dos . uno)^n))((x- tres)^n)
  • ((((3.7) en el grado n)(n al cuadrado )) dividir por ((2.1) en el grado n))((x menos 3) en el grado n)
  • ((((tres . siete) en el grado n)(n en el grado dos)) dividir por ((dos . uno) en el grado n))((x menos tres) en el grado n)
  • ((((3.7)n)(n2))/((2.1)n))((x-3)n)
  • 3.7nn2/2.1nx-3n
  • ((((3.7)^n)(n²))/((2.1)^n))((x-3)^n)
  • ((((3.7) en el grado n)(n en el grado 2))/((2.1) en el grado n))((x-3) en el grado n)
  • 3.7^nn^2/2.1^nx-3^n
  • ((((3.7)^n)(n^2)) dividir por ((2.1)^n))((x-3)^n)
  • Expresiones semejantes

  • ((((3.7)^n)(n^2))/((2.1)^n))((x+3)^n)

Suma de la serie ((((3.7)^n)(n^2))/((2.1)^n))((x-3)^n)



=

Solución

Ha introducido [src]
   oo                    
______                   
\     `                  
 \          n            
  \     /37\   2         
   \    |--| *n          
    \   \10/            n
     )  --------*(x - 3) 
    /        n           
   /     /21\            
  /      |--|            
 /       \10/            
/_____,                  
 n = 1                   
$$\sum_{n=1}^{\infty} \frac{\left(\frac{37}{10}\right)^{n} n^{2}}{\left(\frac{21}{10}\right)^{n}} \left(x - 3\right)^{n}$$
Sum((((37/10)^n*n^2)/(21/10)^n)*(x - 3)^n, (n, 1, oo))
Radio de convergencia de la serie de potencias
Se da una serie:
$$\frac{\left(\frac{37}{10}\right)^{n} n^{2}}{\left(\frac{21}{10}\right)^{n}} \left(x - 3\right)^{n}$$
Es la serie del tipo
$$a_{n} \left(c x - x_{0}\right)^{d n}$$
- serie de potencias.
El radio de convergencia de la serie de potencias puede calcularse por la fórmula:
$$R^{d} = \frac{x_{0} + \lim_{n \to \infty} \left|{\frac{a_{n}}{a_{n + 1}}}\right|}{c}$$
En nuestro caso
$$a_{n} = \left(\frac{21}{10}\right)^{- n} \left(\frac{37}{10}\right)^{n} n^{2}$$
y
$$x_{0} = 3$$
,
$$d = 1$$
,
$$c = 1$$
entonces
$$R = 3 + \lim_{n \to \infty}\left(\frac{\left(\frac{21}{10}\right)^{- n} \left(\frac{21}{10}\right)^{n + 1} \left(\frac{37}{10}\right)^{n} \left(\frac{37}{10}\right)^{- n - 1} n^{2}}{\left(n + 1\right)^{2}}\right)$$
Tomamos como el límite
hallamos
$$R^{1} = \frac{132}{37}$$
$$R^{1} = 3.56756756756757$$
$$R = 3.56756756756757$$
Respuesta [src]
/  /  37   37*x\ /  30   37*x\                       
|  |- -- + ----|*|- -- + ----|                       
|  \  7     21 / \  7     21 /        37*|-3 + x|    
|  ---------------------------    for ----------- < 1
|                     3                    21        
|          /44   37*x\                               
|          |-- - ----|                               
|          \7     21 /                               
|                                                    
<  oo                                                
|____                                                
|\   `                                               
| \        -n     n                                  
|  \   /21\   /37\   2         n                     
|  /   |--|  *|--| *n *(-3 + x)        otherwise     
| /    \10/   \10/                                   
|/___,                                               
|n = 1                                               
\                                                    
$$\begin{cases} \frac{\left(\frac{37 x}{21} - \frac{37}{7}\right) \left(\frac{37 x}{21} - \frac{30}{7}\right)}{\left(\frac{44}{7} - \frac{37 x}{21}\right)^{3}} & \text{for}\: \frac{37 \left|{x - 3}\right|}{21} < 1 \\\sum_{n=1}^{\infty} \left(\frac{21}{10}\right)^{- n} \left(\frac{37}{10}\right)^{n} n^{2} \left(x - 3\right)^{n} & \text{otherwise} \end{cases}$$
Piecewise(((-37/7 + 37*x/21)*(-30/7 + 37*x/21)/(44/7 - 37*x/21)^3, 37*|-3 + x|/21 < 1), (Sum((21/10)^(-n)*(37/10)^n*n^2*(-3 + x)^n, (n, 1, oo)), True))

    Ejemplos de hallazgo de la suma de la serie